LSAT and Law School Admissions Forum

Get expert LSAT preparation and law school admissions advice from PowerScore Test Preparation.

User avatar
 Dave Killoran
PowerScore Staff
  • PowerScore Staff
  • Posts: 5852
  • Joined: Mar 25, 2011
|
#87684
Complete Question Explanation
(The complete setup for this game can be found here: lsat/viewtopic.php?f=151&t=8310)

The correct answer choice is (C)


If S departs third, then only R or T can depart fourth. With S and Q placed, and R precluded from departing first due to its domestic status, only P or T can depart first. This information leads to the following basic setup:

G3-Q17.png

S must depart before R or T but there is not enough room for S to depart before both R and T. Thus, answer choice (C) cannot occur and is correct.

Get the most out of your LSAT Prep Plus subscription.

Analyze and track your performance with our Testing and Analytics Package.